7
$\begingroup$

Suppose $p_n$ is $n$-th prime, $g_n=p_{n+1}-p_n$ is the corresponding prime gap. What is the highest number $C$, such that $p_N>C$ can be proven for $N=\min\{n\mid g_n\geq 1.609\cdot 10^{18}\}$.

Motivation: I've read about Goldbach's weak conjecture. The number $C$ above is obvious lower bound for the first odd number, which does not admit a representation as a sum of three primes, which follows from check of Goldbach's conjecture up to $1.609\cdot 10^{18}$, which is done already by computers. I just want to know, how big is it.

$\endgroup$
4
  • 1
    $\begingroup$ Is there any particular reason for asking? $\endgroup$
    – David Roberts
    Oct 5, 2010 at 11:26
  • $\begingroup$ What does $P_N$ mean? $\endgroup$ Oct 5, 2010 at 11:50
  • $\begingroup$ It should be $p_N$ instead of $P_N$. Corrected. Thank you. $\endgroup$
    – Fiktor
    Oct 5, 2010 at 12:24
  • 2
    $\begingroup$ Marek Wolf has recently presented some statistics and conjectures regarding prime gaps. Although not proven, his guesses are in accord with observed data and others predictions. Check it out. arxiv.org/abs/1102.0481v1 . Gerhard "Ask Me About System Design" Paseman, 2011.02.08 $\endgroup$ Feb 8, 2011 at 19:11

3 Answers 3

8
$\begingroup$

The title and body ask different questions, so I'll address both.

A reasonable estimate for the first prime gap of length $L$ is $e^{\sqrt L}$, so no prime gap this large would be expected below $\exp(1.268\cdot10^9)$, a 550,886,759-digit number.

As for a lower bound, Dusart 2010 [1] shows that for $x\ge396,738$, there is a prime between x and $x\left(1+\frac{1}{25\log^2x}\right)$, so 113353896002617492536754 (about $1.13\cdot10^{23}$) is a lower bound.

Under the Riemann hypothesis ([2]), the bound can be improved to 15373988432858515871940264945439 (about $1.5\cdot10^{31}$).

[1] http://arxiv.org/abs/1002.0442

[2] Lowell Schoenfeld, 'Sharper Bounds for the Chebyshev Functions $\theta(x)$ and $\psi(x)$. II'. Mathematics of Computation, Vol 30, No 134 (Apr 1976), pp. 337-360.

$\endgroup$
13
$\begingroup$

From the Wikipedia page on Bertrand's postulate, Dusart (1998) showed that for all $x > 3275$, there exists a prime between $x$ and $x \left( 1 + \frac{1}{2 \ln^2 x} \right)$. You are looking for the largest $N$ such that $\frac{N}{2\ln^2 N} < 1.609 \cdot 10^{18}$.

A quick computation yields about $8.193\cdot 10^{21}$.

$\endgroup$
1
  • 5
    $\begingroup$ There is an earlier estimate on the same page that yields a better lower bound. It says that for $x > 2010670$ there is a prime between $x$ and $16598x/16597$, so the lower bound is at least $2.67 \cdot 10^{22}$. $\endgroup$
    – S. Carnahan
    Oct 5, 2010 at 12:17
8
$\begingroup$

Well, Wikipedia's page on Bertrand's postulate which Scott referred to in his answer does not cite the strongest estimates on this problem. The paper of Olivier Ramaré and Yannick Saouter MR 2004a:11095 "Short effective intervals containing primes. J. Number Theory 98 (2003), no. 1, 10–33 yield stronger result. They prove that the interval $[x(1-1/\Delta), x]$ contains a prime for $x \geq 10 726 905 041$ and $\Delta=28 313 999$. In fact they have a table that gives even stronger result in the relevant range for this problem. If we look at Table 1 from their paper we get that for $x \geq e^{60}$ we can choose $\Delta=209 267 308$. We thus need to determine $x$ so that $$ \frac x {209 267 308} <1.609 \cdot 10^{18}$$ Computation shows that $x<3.367\cdot 10^{26}$. Since $\log(3.367\cdot 10^{26})=61.1>60$ this is permissable. This gives a better estimate than Charles and Scott's answers above.

It should be remarked that already Ramaré-Saouter used these estimates for the Ternary Goldbach problem. However due to a shorter range where the Goldbach problem had been checked at that time they did get the shorter range $1.13256\cdot 10^{22}$ (Corrollary 1) than using the recent computer checked bounds for the Goldbach problem.

$\endgroup$
1
  • 1
    $\begingroup$ Excellent! You get my +1. @Fiktor should consider de-selecting my answer in favor of yours. $\endgroup$
    – Charles
    Feb 20, 2011 at 6:20

Your Answer

By clicking “Post Your Answer”, you agree to our terms of service and acknowledge you have read our privacy policy.

Not the answer you're looking for? Browse other questions tagged or ask your own question.